Diễn Đàn MathScopeDiễn Đàn MathScope
  Diễn Đàn MathScope
Ghi Danh Hỏi/Ðáp Thành Viên Social Groups Lịch Ðánh Dấu Ðã Ðọc

Go Back   Diễn Đàn MathScope > Sơ Cấp > Việt Nam và IMO > 2011

News & Announcements

Ngoài một số quy định đã được nêu trong phần Quy định của Ghi Danh , mọi người tranh thủ bỏ ra 5 phút để đọc thêm một số Quy định sau để khỏi bị treo nick ở MathScope nhé !

* Nội quy MathScope.Org

* Một số quy định chung !

* Quy định về việc viết bài trong diễn đàn MathScope

* Nếu bạn muốn gia nhập đội ngũ BQT thì vui lòng tham gia tại đây

* Những câu hỏi thường gặp

* Về việc viết bài trong Box Đại học và Sau đại học


Trả lời Gởi Ðề Tài Mới
 
Ðiều Chỉnh Xếp Bài
Old 11-01-2011, 12:18 PM   #16
Evarist Galois
+Thành Viên+
 
Evarist Galois's Avatar
 
Tham gia ngày: Nov 2009
Đến từ: Từ A0 đến FTU
Bài gởi: 320
Thanks: 57
Thanked 180 Times in 95 Posts
Bài hình a đã từng có trên diễn đàn: [Only registered and activated users can see links. ]
Câu b thì khỏi nói tính tỉ số CM/MB=menelaus và hệ thức lượng. Bài hình như bài lớp 9 vậy.
Bài BDT quy nạp là ra
[RIGHT][I][B]Nguồn: MathScope.ORG[/B][/I][/RIGHT]
 
__________________

thay đổi nội dung bởi: Evarist Galois, 11-01-2011 lúc 01:01 PM
Evarist Galois is offline   Trả Lời Với Trích Dẫn
The Following 6 Users Say Thank You to Evarist Galois For This Useful Post:
huynhcongbang (12-01-2011), IMO 2010 (13-01-2011), Ino_chan (06-03-2011), lehuu (12-01-2011), lucatony1234 (13-01-2011), Thuong_19 (05-03-2011)
Old 11-01-2011, 12:21 PM   #17
novae
+Thành Viên Danh Dự+
 
novae's Avatar
 
Tham gia ngày: Jul 2010
Đến từ: Event horizon
Bài gởi: 2,453
Thanks: 53
Thanked 3,057 Times in 1,288 Posts
Tính cái tỉ số $\frac{PC}{CA}=\tan^2\alpha $ với $\alpha=\widehat{BAP} $
Áp dụng Van Aubel ra $\frac{PM}{MO}=2\tan^2 \alpha $
Từ đó thẳng tiến mà làm
[RIGHT][I][B]Nguồn: MathScope.ORG[/B][/I][/RIGHT]
 
__________________
M.
novae is offline   Trả Lời Với Trích Dẫn
The Following 3 Users Say Thank You to novae For This Useful Post:
IMO 2010 (13-01-2011), Ino_chan (06-03-2011), lehuu (12-01-2011)
Old 11-01-2011, 12:24 PM   #18
khicon
+Thành Viên+
 
Tham gia ngày: Aug 2009
Bài gởi: 73
Thanks: 77
Thanked 19 Times in 14 Posts
Trích:
Nguyên văn bởi novae View Post
Tính cái tỉ số $\frac{PC}{CA}=\tan^2\alpha $ với $\alpha=\widehat{BAP} $
Áp dụng Van Aubel ra $\frac{PM}{MO}=2\tan^2 \alpha $
Từ đó thẳng tiến mà làm
anh Minh xem hộ em ý tưởng bài tổ hợp như em làm ở trên có đúng không ạ??
[RIGHT][I][B]Nguồn: MathScope.ORG[/B][/I][/RIGHT]
 
khicon is offline   Trả Lời Với Trích Dẫn
Old 11-01-2011, 12:28 PM   #19
novae
+Thành Viên Danh Dự+
 
novae's Avatar
 
Tham gia ngày: Jul 2010
Đến từ: Event horizon
Bài gởi: 2,453
Thanks: 53
Thanked 3,057 Times in 1,288 Posts
Trích:
Nguyên văn bởi khicon View Post
anh Minh xem hộ em ý tưởng bài tổ hợp như em làm ở trên có đúng không ạ??
Lấy hình thoi có 1 góc 60 là thấy sai ngay
[RIGHT][I][B]Nguồn: MathScope.ORG[/B][/I][/RIGHT]
 
__________________
M.
novae is offline   Trả Lời Với Trích Dẫn
The Following User Says Thank You to novae For This Useful Post:
khicon (11-01-2011)
Old 11-01-2011, 12:28 PM   #20
n.v.thanh
Moderator
 
n.v.thanh's Avatar
 
Tham gia ngày: Nov 2009
Bài gởi: 2,849
Thanks: 2,980
Thanked 2,537 Times in 1,008 Posts
Trích:
Nguyên văn bởi thangtoancvp View Post
Câu 1. Chứng minh bằng quy nạp:
Ta có $(\frac{x+1}{2})^{2n+3}=(\frac{x+1}{2})^{2n+1} (\frac{x+1}{2})^2\ge \frac{x^{n+1}(x^{n+2}+1)}{x^{n+1}+1}.(\frac{x+1}{2 })^2 $.
Ta sẽ chứngminh $\frac{x^{n+1}(x^{n+2}+1)}{x^{n+1}+1}.(\frac{x+1}{2 })^2\ge \frac{x^{n}(x^{n+1}+1)}{x^{n}+1}.(\frac{x+1}{2})^2 $ bất đẳng thức này tương đương với: $(x^{n+1}-1)^2(x-1)^2\ge 0 $
Bài này em không làm được
Như trên nhưng cái bất đẳng thức cần cm em phá hết ra,cuống quá ghi
nó tương đương
$(x-1)^2.A \geq 0 $.....
luôn đúng
.Bình tĩnh chia đa thức thì đã.Chắc dc 4/5.
.Tình hình khtn khá ổn nhé
Đề trông dễ chịu nhứng sao ai cũng zZz thế này?
[RIGHT][I][B]Nguồn: MathScope.ORG[/B][/I][/RIGHT]
 

thay đổi nội dung bởi: n.v.thanh, 11-01-2011 lúc 12:33 PM
n.v.thanh is offline   Trả Lời Với Trích Dẫn
Old 11-01-2011, 12:34 PM   #21
vinhhop.qt
+Thành Viên+
 
Tham gia ngày: Mar 2010
Bài gởi: 86
Thanks: 44
Thanked 70 Times in 34 Posts
Trích:
Nguyên văn bởi novae View Post
Bài 4 (5.0 điểm)
Cho ngũ giác lồi $ABCDE $ có các cạnh và 2 đường chéo $AC,AD $ có độ dài không vượt quá $\sqrt3 $. Trong ngũ giác lồi lấy $2011 $ điểm phân biệt bất kì. Chứng minh rằng tồn tại một hình tròn đơn vị có tâm nằm trên cạnh của ngũ giác lồi $ABCDE $ và chứa ít nhất $403 $ điểm trong số $2011 $ điểm đã cho
Bài này sử dụng ý tưởng "một tam giác cạnh không quá $\sqrt{3} $ thì phủ được bằng ba hình tròn đơn vị tâm tại các đỉnh của tam giác".
Chứng minh điều này như sau: Giả sử ngược lại, có điểm M không được phủ bởi 3 hình tròn nói trên. Thì vì khoảng cách từ giao điểm của hai đường tròn bất kì đến các cạnh lớn hơn hoặc bằng 1/2, nên khoảng cách từ M đến các cạnh của tam giác đều lớn hơn 1/2. Suy ra diện tích của tam giác lơn hơn $\frac{a+b+c}{4} $. Mặt khác, theo công thức Heron, diện tích tam giác ko quá $\frac{1}{4}\sqrt{(a+b+c)abc}\le \frac{1}{4}(a+b+c) $ với a,b,c không vượt quá $\sqrt{3} $. Từ đó dẫn đến điều vô lí.
Sau đó xét 5 hình tròn đơn vị có tâm là các đỉnh của ngũ giác đã cho, theo nguyên lí Dirichllet, ta có điều phải chứng minh.
[RIGHT][I][B]Nguồn: MathScope.ORG[/B][/I][/RIGHT]
 
vinhhop.qt is offline   Trả Lời Với Trích Dẫn
The Following 5 Users Say Thank You to vinhhop.qt For This Useful Post:
huynhcongbang (12-01-2011), IMO 2010 (13-01-2011), lehuu (12-01-2011), lucatony1234 (13-01-2011), tranvuxuannhat (12-01-2011)
Old 11-01-2011, 12:34 PM   #22
nbkschool
+Thành Viên+
 
nbkschool's Avatar
 
Tham gia ngày: Dec 2007
Đến từ: SMU Residence @Prinsep Hostel, 83 Prinsep Street, Singapore
Bài gởi: 400
Thanks: 72
Thanked 223 Times in 106 Posts
Tạch rồi.Làm đc mỗi 2,3 mà 3.2 làm nhầm.Ôn thi ĐH thôi!
[RIGHT][I][B]Nguồn: MathScope.ORG[/B][/I][/RIGHT]
 
__________________
"Apres moi,le deluge"
nbkschool is offline   Trả Lời Với Trích Dẫn
The Following User Says Thank You to nbkschool For This Useful Post:
binladen93 (11-01-2011)
Old 11-01-2011, 12:35 PM   #23
hophinhan_LHP
+Thành Viên+
 
hophinhan_LHP's Avatar
 
Tham gia ngày: Dec 2008
Đến từ: 12CT_THPT Chuyên LHP_TPHCM
Bài gởi: 226
Thanks: 199
Thanked 136 Times in 81 Posts
Trích:
Nguyên văn bởi nbkschool View Post
Tạch rồi.Làm đc mỗi 2,3 mà 3.2 làm nhầm.Ôn thi ĐH thôi!
tui cũng đại học luôn, lăng tăng gì nửa
[RIGHT][I][B]Nguồn: MathScope.ORG[/B][/I][/RIGHT]
 
__________________
ĐẠI HỌC THÔI !!!
hophinhan_LHP is offline   Trả Lời Với Trích Dẫn
Old 11-01-2011, 12:39 PM   #24
n.v.thanh
Moderator
 
n.v.thanh's Avatar
 
Tham gia ngày: Nov 2009
Bài gởi: 2,849
Thanks: 2,980
Thanked 2,537 Times in 1,008 Posts
Chính xác.Câu b tọa độ đẹp cực kì.

@@nbkschool:
Ngày mai mới là đất của anh mà.

@@ toanlc_gift
ban đầu em cũng nghĩ ngược dấu nên ko làm...Sau mới chém vào,đúng mới may
Tại 2 vế bậc lệch nhau nhiều nên mới ko ngược dấu.Cái bài 1 nhẹ chán.
Nói chung là ngày 1 đề dễ chịu thật
[RIGHT][I][B]Nguồn: MathScope.ORG[/B][/I][/RIGHT]
 

thay đổi nội dung bởi: n.v.thanh, 11-01-2011 lúc 12:45 PM
n.v.thanh is offline   Trả Lời Với Trích Dẫn
Old 11-01-2011, 12:41 PM   #25
IMO
+Thành Viên+
 
IMO's Avatar
 
Tham gia ngày: Sep 2009
Đến từ: VIỆT NAM
Bài gởi: 120
Thanks: 0
Thanked 110 Times in 37 Posts
Trích:
Nguyên văn bởi huy2710 View Post
Đề kì này trông cũng dễ chịu (chưa biết ngày mai ra sao )
Ngồi hoài mà chả làm được câu hình mới ngu
Mai chắc bỏ trắng luôn quá
Ở đây có ai người Nình Bình ko vậy?
Tình hình làm bài của đội Nình Bình thế nào nhỉ?
[RIGHT][I][B]Nguồn: MathScope.ORG[/B][/I][/RIGHT]
 
__________________
Gia Sư Trực Tuyến Việt Nam
Http://GSTT.VN
IMO is offline   Trả Lời Với Trích Dẫn
Old 11-01-2011, 12:51 PM   #26
nbkschool
+Thành Viên+
 
nbkschool's Avatar
 
Tham gia ngày: Dec 2007
Đến từ: SMU Residence @Prinsep Hostel, 83 Prinsep Street, Singapore
Bài gởi: 400
Thanks: 72
Thanked 223 Times in 106 Posts
Đồng Nai Hòa 3 bài Thành lớn 2 bài Thành bé hình như cũng 2.
Mai làm gì nữa.Biết thừa là bài khó thì mình không làm được,bài dễ cũng...ko làm được tất.
[RIGHT][I][B]Nguồn: MathScope.ORG[/B][/I][/RIGHT]
 
__________________
"Apres moi,le deluge"
nbkschool is offline   Trả Lời Với Trích Dẫn
The Following User Says Thank You to nbkschool For This Useful Post:
n.v.thanh (11-01-2011)
Old 11-01-2011, 12:51 PM   #27
view
+Thành Viên+
 
view's Avatar
 
Tham gia ngày: Sep 2010
Đến từ: Nghe An
Bài gởi: 7
Thanks: 7
Thanked 2 Times in 1 Post
à cho em hỏi tình hình anh Thắng bên tổng hợp làm thế nào
còn anh Thanh ăn ngon ngủ yên ko
[RIGHT][I][B]Nguồn: MathScope.ORG[/B][/I][/RIGHT]
 
__________________
Ta đang tìm mình giữa các vì sao
view is offline   Trả Lời Với Trích Dẫn
Old 11-01-2011, 12:57 PM   #28
sonltv_94
+Thành Viên+
 
sonltv_94's Avatar
 
Tham gia ngày: Aug 2009
Đến từ: Biên Hòa Đồng Nai
Bài gởi: 149
Thanks: 29
Thanked 139 Times in 85 Posts
Xong.Anh còn nói thế thì Đồng Nai ai đủ sức để gánh vác 2% hy vọng ở TST
[RIGHT][I][B]Nguồn: MathScope.ORG[/B][/I][/RIGHT]
 
__________________
Vĩnh biệt Toán,vĩnh biệt Mathscope....
sonltv_94 is offline   Trả Lời Với Trích Dẫn
Old 11-01-2011, 01:03 PM   #29
n.v.thanh
Moderator
 
n.v.thanh's Avatar
 
Tham gia ngày: Nov 2009
Bài gởi: 2,849
Thanks: 2,980
Thanked 2,537 Times in 1,008 Posts
Trích:
Nguyên văn bởi view View Post
à cho em hỏi tình hình anh Thắng bên tổng hợp làm thế nào
còn anh Thanh ăn ngon ngủ yên ko
anh 2,5.Thắng nó 3 chú ạ.a Long 4.a Rực gần 4,còn bài dãy đang dở tay hay sao ý.
[RIGHT][I][B]Nguồn: MathScope.ORG[/B][/I][/RIGHT]
 
n.v.thanh is offline   Trả Lời Với Trích Dẫn
Old 11-01-2011, 01:08 PM   #30
newbie
+Thành Viên+
 
Tham gia ngày: Jun 2009
Bài gởi: 266
Thanks: 17
Thanked 164 Times in 84 Posts
Đề này ngồi ngoài làm thấy nhẹ nhàng
Bài 1 :
Sử dụng quy nạp ta đưa về cm :
$ \frac{x^{n+1}(x^{n+2}+1)}{x^{n+1}+1} \le \frac{( x+1)^2}{4} .\frac{x^{n}(x^{n+1}+1)}{x^{n}+1} $
$\frac{( x^n+1)(x^{n+2}+1)}{(x^{n+1}+1)^2} \le \frac{ (x+1)^2 }{4x} $
$ \frac{ x^n(x-1)^2}{ ( x^{n+1}+1)^2 } \le \frac{ (x-1)^2}{4x} $
$4x^{n+1} \le (x^{n+1}+1)^2 $
hiển nhiên đúng
Bài 2:
$x_{n+1}=\frac{2(n+1)}{n^2}( x_n+ \sum^{n-1}_{i=1}x_i )=\frac{2(n+1)}{n^2}( x_n + \frac{(n-1)^2}{2n}x_n )=\frac{ n+1 }{n}.\frac{n^2+1}{n^2} x_n $
Vậy ta suy đuợc 2 điều sau :
+ $y_{n+1}=\frac{ n^2+n+1}{n^2}. \frac{x_n}{n} $
+ $\frac{x_{n+1}}{n+1}=\prod^n_{i=1} (1+\frac{1}{i^2}) $

Việc còn lại là chứng minh $z_n= \prod^n_{i=1} (1+\frac{1}{i^2}) $
Mà bài này thì ai từng học qua giới hạn đều gặp cả

Bài 3
a) Pascal
b)Không cần quan tâm đến mấy điểm E,D thì thấy bài này đủ hết các yếu tố để xài Ceva .Cách hình học thì chưa nghĩ ra
Bài 4
Ý tưởng lộ rõ khi nhắc đến 2 đường chéo $AC $ và $AD $.Nối 2 đường chéo ấy lại , ta chia ngũ giác ra làm 3 tam giác .Từ đây xài bài toán phụ : Tam giác có 3 cạnh nhỏ hơn $\sqrt{3} $ thì tam giác ấy bị phụ kín bởi các hình tròn đơn vị có tâm ở các đỉnh
[RIGHT][I][B]Nguồn: MathScope.ORG[/B][/I][/RIGHT]
 

thay đổi nội dung bởi: newbie, 11-01-2011 lúc 01:11 PM
newbie is offline   Trả Lời Với Trích Dẫn
The Following 13 Users Say Thank You to newbie For This Useful Post:
binladen93 (11-01-2011), huynhcongbang (12-01-2011), IMO 2010 (13-01-2011), letientai (11-01-2011), lk_95 (11-01-2011), long_chau2010 (11-01-2011), lucatony1234 (13-01-2011), manhnguyen94 (11-01-2011), n.v.thanh (11-01-2011), thanh_kha (11-01-2011), toanlc_gift (11-01-2011), tuan119 (11-01-2011), Unknowing (11-01-2011)
Trả lời Gởi Ðề Tài Mới

Bookmarks

Tags
2010-2011, hsg quốc gia

Ðiều Chỉnh
Xếp Bài

Quuyền Hạn Của Bạn
You may not post new threads
You may not post replies
You may not post attachments
You may not edit your posts

BB code is Mở
Smilies đang Mở
[IMG] đang Mở
HTML đang Tắt

Chuyển đến


Múi giờ GMT. Hiện tại là 01:14 AM.


Powered by: vBulletin Copyright ©2000-2024, Jelsoft Enterprises Ltd.
Inactive Reminders By mathscope.org
[page compression: 107.15 k/123.29 k (13.09%)]